1952 AHSME Problems/Problem 17

Revision as of 01:44, 3 January 2014 by Throwaway1489 (talk | contribs) (Created page with "== Problem== A merchant bought some goods at a discount of <math> 20\% </math> of the list price. He wants to mark them at such a price that he can give a discount of <math> 20\...")
(diff) ← Older revision | Latest revision (diff) | Newer revision → (diff)

Problem

A merchant bought some goods at a discount of $20\%$ of the list price. He wants to mark them at such a price that he can give a discount of $20\%$ of the marked price and still make a profit of $20\%$ of the selling price. The per cent of the list price at which he should mark them is:

$\textbf{(A) \ }20  \qquad \textbf{(B) \ }100 \qquad \textbf{(C) \ }125 \qquad \textbf{(D) \ }80 \qquad \textbf{(E) \ }120$

Solution

Let $C$ represent the cost of the goods, and let $L$, $S$, and $M$ represent the list, selling, and marked prices of the goods, respectively. Hence, we have three equations, which we need to manipulate in order to relate $M$ and $L$:

$C=\frac{4}{5}L$

$S=C+\frac{1}{5}S$

$S=\frac{4}{5}M$

We find that $M=\frac{5}{4}L$. Hence, the percent of the list price which should be marked is $\boxed{\textbf{(C)}\ 125}$.

See also

1952 AHSC (ProblemsAnswer KeyResources)
Preceded by
Problem 16
Followed by
Problem 18
1 2 3 4 5 6 7 8 9 10 11 12 13 14 15 16 17 18 19 20 21 22 23 24 25 26 27 28 29 30 31 32 33 34 35 36 37 38 39 40 41 42 43 44 45 46 47 48 49 50
All AHSME Problems and Solutions

The problems on this page are copyrighted by the Mathematical Association of America's American Mathematics Competitions. AMC logo.png